Fonctions non constructibles et résultats anormaux

10

Dans le livre Arora-Barak, dans la définition des fonctions constructibles dans le temps, il est dit que l'utilisation de fonctions qui ne sont pas constructibles dans le temps peut conduire à des "résultats anormaux". Quelqu'un at-il un exemple d'un tel "résultat anormal"? J'ai entendu en particulier qu'il pourrait exister des fonctions telles que le théorème de la hiérarchie temporelle ne tient pas, est-ce que quelqu'un a un exemple de telles fonctions? Y a-t-il quelque chose à ce sujet quelque part dans la littérature?

Pascal
la source
3
Avez - vous eu un coup d' œil ceux - ci: math.stackexchange.com/questions/51082/... et cstheory.stackexchange.com/questions/992/... ?
Jukka Suomela
@JukkaSuomela: Oui, mais elles concernent les fonctions constructibles temps / espace et pourquoi elles sont utiles.
Pascal

Réponses:

11

Borodin's Gap Theorem : Pour chaque fonction calculable totale , il existe une fonction calculable totale telle que .t D T I M E [ g ( t ( n ) ) ] = D T I M E [ t ( n ) ]g(n)ntDTIME[g(t(n))]=DTIME[t(n)]

En fait, cela vaut pour toute mesure de complexité Blum à la place de .DTIME

Voir aussi la page wikipedia et les références qui s'y trouvent.

Joshua Grochow
la source
6

Étant donné que l'article Wikipedia ne donne pas la preuve et que le document est sur ACM DL, j'ai pensé qu'il pourrait être utile de publier la preuve ici:

THÉORÈME 3.7. (Théorème des lacunes).

Soit une mesure de complexité, une fonction récursive non décroissante telle que . Il existe alors une fonction récursive croissante telle que les fonctions calculables de la mesure de complexité sont les mêmes que les fonctions calculables de la mesure de complexité .g x , g ( x ) x t t g tΦgx,g(x)xttgt

PREUVE.

Définissez comme suit:t

t ( n ) : = μ k > t ( n - 1 ) : i n , ( Φ i ( n ) < k Φ i ( n ) > g ( k ) )

t(0):=1
t(n):=μk>t(n1):in,(Φi(n)<kΦi(n)>g(k))
  1. pour tout , il y a un , car pour tout :k i nnkjen

    une. si n'est pas défini, alors , etk , Φ i ( n ) > g ( k )Φje(n)k,Φje(n)>g(k)

    b. si défini alors .k , Φ i ( n ) < kΦje(n)k,Φje(n)<k

  2. Φ Φ i ( n ) < k Φ i ( n ) > g ( k )k peut être trouvé récursivement, car est une mesure de complexité et donc et sont des prédicats récursifs.ΦΦje(n)<kΦje(n)>g(k)

  3. n i Φ i ( n ) < t ( n ) Φ i ( n ) > g t ( n )t satisfait le théorème, car implique que ou .njeΦje(n)<t(n)Φje(n)>gt(n)

QED.

Nous observons qu'un arbitrairement grand peut être trouvé pour satisfaire le théorème 3.7. Supposons que nous voulons , puis définissonst ( n ) > r ( n )tt(n)>r(n)

t ( n ) : = μ k > m a x { t ( n - 1 ) , r ( n ) } :

t(0): =r(0)+1
t(n): =μk>muneX{t(n-1),r(n)}:

(d'après Allan Borodin, " La complexité informatique et l'existence de lacunes de complexité ", JACM 1972, avec de légères modifications.)

Kaveh
la source
t(n)kng(k)k